luckyzrjz
Thanks Received: 0
Vinny Gambini
Vinny Gambini
 
Posts: 5
Joined: September 19th, 2011
 
 
 

Q19 - Doctor: Medical researchers recently examined

by luckyzrjz Thu Sep 22, 2011 2:23 am

Can anyone help to explain why choice (A) is wrong? Thanks a lot!
User avatar
 
gilad.bendheim
Thanks Received: 21
Jackie Chiles
Jackie Chiles
 
Posts: 31
Joined: August 20th, 2011
 
 
 

Re: Q19 - Doctor: Medical researchers recently examined

by gilad.bendheim Fri Sep 23, 2011 9:57 am

Hi - I'm not sure, but I'll give it a go.

Choice (A) is tricky. But the problem may be that the stimulas never talks about a case where 'the factor [bulged disks] is not present.' We dont know anything about the cause of back pain, other than the study which seems to say that slipped/bulged disks aren't ENOUGH to cause back pain. We dont necessarily have any reason to think that a person can have serious back pain without having a slipped disk. All we know is that someone can have a slipped disk without having serious back pain. That is, that perhaps a slipped disk is a necessary but not sufficient cause for back pain (notice that choice (B) basically says this).

Choice (A) is forcing us to make another assumption beyond the purview of the stimulas. In assumption and weaken questions, it is important to make sure that the answer choice is actually dealing with the premise as it is laid out, and not our own immediate calculations based on the premise (this is how they want to trick us!). So in a question that is based on a premise about when back pain DOES NOT occur, we have to avoid answers whose premise is about when back pain DOES occur.

Again, not positive about this one, but maybe it helps clear things up.
User avatar
 
LSAT-Chang
Thanks Received: 38
Atticus Finch
Atticus Finch
 
Posts: 479
Joined: June 03rd, 2011
 
 
trophy
Most Thankful
trophy
First Responder
 

Re: Q19 - Medical researchers recently examined

by LSAT-Chang Tue Sep 27, 2011 3:50 pm

I didn't even understand what was going on in this argument.
What exactly is the "flaw" that is being committed here??? I felt like it had to do something with the whole necessary/sufficient stuff, so I narrowed it down to (A) and (B) and just picked (A) and moved on since I was doing the full section timed. I was trying to diagram this stuff out, but it wasn't leading me anywhere close "reversed logic" or "negated logic" so wasn't sure what the flaw even was. Can someone please provide any help? :(
 
luckyzrjz
Thanks Received: 0
Vinny Gambini
Vinny Gambini
 
Posts: 5
Joined: September 19th, 2011
 
 
 

Re: Q19 - Medical researchers recently examined

by luckyzrjz Fri Sep 30, 2011 6:17 pm

I read the argument all over again. This time the reason why choice A is wrong is clear. The doctor says "half of the members of the group" got the slipped disks. So if the disk problem is sufficient to cause pain, these people should feel the pain!!! So it obviously is not sufficient to produce the effect. That is why choice A is wrong. But why is this kind of problem often be blamed for the serious back pain? Maybe because this slipped disk problem is one of several important factors which together can lead to the severe back pain. So that is what B is trying to say. And I think gilad.bendheim's thoughts also help to clarify the issue here.
User avatar
 
ManhattanPrepLSAT1
Thanks Received: 1909
Atticus Finch
Atticus Finch
 
Posts: 2851
Joined: October 07th, 2009
 
This post thanked 4 times.
 
 

Re: Q19 - Medical researchers recently examined

by ManhattanPrepLSAT1 Sat Oct 08, 2011 3:42 pm

changsoyeon Wrote:I didn't even understand what was going on in this argument.
What exactly is the "flaw" that is being committed here??? I felt like it had to do something with the whole necessary/sufficient stuff, so I narrowed it down to (A) and (B) and just picked (A) and moved on since I was doing the full section timed. I was trying to diagram this stuff out, but it wasn't leading me anywhere close "reversed logic" or "negated logic" so wasn't sure what the flaw even was. Can someone please provide any help?

The reason why you had such a difficult time diagramming this question is that it is not one that notation is going to be very effective. The reason is that the argument is not only about conditional relationships. It's also about causation. And when an argument moves from correlation to causation, it's going to be tougher to put that into notation.

We can see that causation is implied in the conclusion when it says "lead to" - words that commonly imply causation.

The argument offers evidence that severs the correlation between slipped disks and back pain. The argument then goes on to conclude that there is no causal connection between a slipped disk and back pain. The issue there is that just because something doesn't always produce a certain result, does not mean that it might not sometimes produce that result. By itself a slipped disk does not appear to cause back pain, but a slipped disk could lead to problems down the road that would cause back pain. Answer choice (B) addresses this issue by pointing out that while alone a slipped disk is not sufficient to cause back pain, it may be a contributing factor. So a slipped disk could lead to back pain when it is accompanied by some other factor.

Let's look at the incorrect answers:

(A) mixes up the evidence. The argument does not suggest in the evidence that back pain can occur in people that do not have a slipped disk, but rather that slipped disks may be present in people who do not have back pain. Back pain being the effect and a slipped disk being the contributing factor.
(C) is not something the argument failed to consider, but is exactly what the argument concluded to be true.
(D) plays off a sampling issue. But there is no reason to suspect that the sample is unrepresentative.
(E) is close but not quite. The conclusion doesn't discuss the likelihood of the factor (a slipped disk) being present, but whether that factor actually contributes to the effect of back pain.

Hope that helps, but let me know if you have further questions on this one!
 
zaidjawed
Thanks Received: 0
Forum Guests
 
Posts: 15
Joined: October 11th, 2012
 
 
 

Re: Q19 - Doctor: Medical researchers recently examined

by zaidjawed Thu Jan 24, 2013 2:58 am

Hey Matt,
Isn't this question in some manner similar to the logic behind Q12 in Preptest 45 Sec 1 regarding dioxin? I see an almost perfect parallel here and used the same technique to weaken this argument.

The way I looked at it was that there could be a supposed cause (the slipped disks and the bulging spines analogous to dioxin) for the stated effect ( serious back pain analogous to changes in hormone concentrations), however the effect is not observed even when the supposed cause is present.

Basically, Cause-------> ~Effect hence ~Cause

The way I reasoned(weakened) this was by considering that either the cause is not present or the effect is present. But the cause is present except that it in this case it could be partly responsible for it.

Do you think that type of reasoning works here?
 
keonheecho
Thanks Received: 0
Elle Woods
Elle Woods
 
Posts: 54
Joined: August 20th, 2014
 
 
 

Re: Q19 - Doctor: Medical researchers recently examined

by keonheecho Sat Jul 11, 2015 10:59 am

Hi, I just wanted to check up on my reasoning here...

Isn't (A) also incorrect because it contradicts the stimulus? (A) says that the factor (bulging disks) may nonetheless be sufficient for the effect (serious back pain). But doesn't the stimulus basically say that some people who have bulging disks don't have the effect occur, which would mean that it is not sufficient right? And also, isn't (A) incorrect because it does not bring up the notion of causality? Even if bulging disks were sufficient for back pain, it wouldn't prove that bulging disks cause back pain. But (B) suggests causality by saying "partly responsible". Please let me know. Thank you
 
frank0478
Thanks Received: 4
Vinny Gambini
Vinny Gambini
 
Posts: 19
Joined: July 21st, 2014
 
This post thanked 1 time.
 
 

Re: Q19 - Doctor: Medical researchers recently examined

by frank0478 Wed Oct 12, 2016 6:28 pm

I tried to simply the thinking a bit:

From the study, you learn that when cause is present (bulge), the effect is not (no pain).
Weird eh? It says usually, people would associate the cause with the effect. Yet here, the effect isn't occurring.
So the author is like, well then the cause must not be a cause at all then!

Maybe. It could be true, if what we take as "usual" ends up being a myth. But, if we assume the premise to be true, that the cause IS usually associated with the effect, then maybe for these guys (no-pain claimers) there is something about them that makes them different and that the cause is only partly contributing to the effect, but something else with these guys (maybe they are all potheads) make them feel none of the effect (pain).

So the author fails to see that for these guys,
the cause is only a partial cause. Hence it is "not in itself sufficient," but "partly responsible... in some instances (like non-potheads)"

Weird thing is the conclusion makes a jump from evidence about the potheads (non-pain) to one about non-potheads (pain). I think another flaw is something like: presumes that result of a study on one group is analogous to another. But here, they want you to focus more on the cause-partial cause part rather than evidence may not necessarily be relevant part.


To reject the rest:
A - when it says a factor that need not be present, we don't know if cause (bulge) need to be present at all. it was never explicitly said that it always has to be there. if you chose this, like I did, that means we were lured into thinking that it has to be. but it doesn't!

B - for these non-pain claimers, it's not sufficient, hence even with the cause the effect does not occur. but it totally could be that it is contributing to the effect, but something about these guys prevents them from experiencing the effect

C - an effect might not occur when the cause exist. it is true. it is descriptively accurate. afterall, we have this cause (bulge) yet no effect. so what? this is just a description, but not one about the flaw.

D - entire population? the author specifically makes a claim about people who do experience such pain. does that sound like the entire population? if you chose this, you made an unwarranted assumption that yes, it does sound like it. but in fact, it doesnt

E - this discuss the probability of cause being present. we aren't even talking about probability though
User avatar
 
ManhattanPrepLSAT1
Thanks Received: 1909
Atticus Finch
Atticus Finch
 
Posts: 2851
Joined: October 07th, 2009
 
 
 

Re: Q19 - Doctor: Medical researchers recently examined

by ManhattanPrepLSAT1 Mon Oct 17, 2016 12:56 pm

Nice work Frank!
 
eds556
Thanks Received: 0
Vinny Gambini
Vinny Gambini
 
Posts: 2
Joined: October 25th, 2017
 
 
 

Re: Q19 - Doctor: Medical researchers recently examined

by eds556 Wed Oct 25, 2017 5:51 pm

why is C wrong
 
JeffW669
Thanks Received: 1
Vinny Gambini
Vinny Gambini
 
Posts: 8
Joined: January 08th, 2018
 
 
 

Re: Q19 - Doctor: Medical researchers recently examined

by JeffW669 Mon Jan 08, 2018 7:53 pm

eds556 Wrote:why is C wrong

What is "An effect that occurs" in this situation? It would be serious back pain. The "effect" being considered in the doctor's argument.
What is "a particular phenomenon" in this situation? It would be a bulging/slipped disk. The "cause" being considered in the doctor's argument.

Now plug these variables in and consider: Does the doctor's argument have anything to do with "serious back pain occurring in the absence of a bulging/slipped disk"? No. The argument concerns the reverse: a bulging/slipped disk occurring in the absence of serious back pain.
User avatar
 
mswang7
Thanks Received: 0
Elle Woods
Elle Woods
 
Posts: 65
Joined: February 27th, 2019
 
 
 

Re: Q19 - Doctor: Medical researchers recently examined

by mswang7 Mon Feb 17, 2020 5:17 pm

Can someone substitute concrete into the abstract answer choices for A & B? I think I am getting the pain/ buldging slipped disks mixed up.
ie I thought A was saying A factor that (pain) that need not be present in order for a certain effect (discs) to arise may nonetheless be sufficient to produce that effect (slipped discs). Am I getting it backwards? I know the conclusion is saying slipped discs cannot lead to serious back pain in people who do experience pain.
What is the best way to correctly substitute the concrete into the abstract in answer choices like these?
If my above substitution of A is correct would that make it the correct answer?
My interpretation of B: A factor (pain) that is not in itself sufficient to produce a certain effect (slipped discs) may nonetheless be partly responsible for that effect (discs) in some instances
 
Laura Damone
Thanks Received: 94
Atticus Finch
Atticus Finch
 
Posts: 468
Joined: February 17th, 2011
 
 
 

Re: Q19 - Doctor: Medical researchers recently examined

by Laura Damone Thu Feb 20, 2020 4:37 pm

Sure!

As you predicted, when you did your substitutions, you switched the concrete elements. We're talking about whether bulging/slipped disks cause back pain, not whether back pain causes bulging/slipped disks!

A: A factor (bulging/slipped disks) that need not be present in order for a certain effect (serious back pain) to arise may nonetheless be sufficient to produce that effect (serious back pain).

B: A factor (bulging/slipped disks) that is not in itself sufficient to produce a certain effect (serious back pain) may nonetheless be partly responsible for that effect (serious back pain) in some instances.

So, which of these sounds more like the argument? Well, the argument shows us folks who do have bulging/slipped disks, but who don't have back pain. That lines up with B - it shows that the bulging/slipped disks aren't sufficient to produce serious back pain. Answer choice A says the opposite - it shows that bulging/slipped disks aren't necessary in order to have back pain.

Then the argument concludes that bulging/slipped disks can't ever lead to serious back pain.

But that's bogus! Just because something doesn't always cause pain doesn't mean it never does. Hangovers don't always cause headaches, but they definitely do sometimes.

This argument therefore overlooks the possibility that, even though bulging/slipped disks don't always lead to serious back pain, bulging/slipped disks may be partly responsible for back pain in some instances. Answer choice B is therefore correct.
Laura Damone
LSAT Content & Curriculum Lead | Manhattan Prep
 
SabrinaM590
Thanks Received: 0
Vinny Gambini
Vinny Gambini
 
Posts: 8
Joined: April 10th, 2020
 
 
 

Re: Q19 - Doctor: Medical researchers recently examined

by SabrinaM590 Fri May 29, 2020 5:53 pm

I am still confused as to why (A) is incorrect, specifically the way it uses the word "sufficient."

Can someone please rephrase this part: " [Slipped or Bulged Disc] may nonetheless be sufficient to produce that effect [Serious Back Pain]."

Is it saying that Bulge or Slipped Discs may not be the cause of Serious Back Pain, but can still be a sufficient condition to produce the effect of SBP (even if SBP is not explicitly stated as a necessary condition in this specific argument)?

--> what is the context with which "sufficient" is used to refer to?

Thank you in advance!